Here's what the NOT flawed version of the stimulus would look like.

(Premise) sound theories AND successful implementation --> lower inflation rate
(Premise) [not] lower inflation rate
___________
(Good conclusion) [not] sound theories AND successful implementation
(Good conclusion with the negation distributed via De Morgan's) not sound theories OR not successful implementation

(Bad conclusion in the stimulus) not sound theories

The argument is flawed because it could be that the theories were fine, just that we sucked at implementing them.

In its abstract form, the flawed argument looks like this:

N and W --> R
/R
___________
/N

(C) matches this form perfectly.

(E) is an attractive wrong answer choice. It's mostly wrong because its logical form does not match:

N --> W and R
/R
___________
/N'

The argument for (E) being better than (C) is that (E) matches the other "mistake" in the argument.

The stimulus argument assumes that "sound" theories = "not far off the mark" theories. True, it does. But, I don't think it's wrong to assume that a "sound" theory is one that's "not far off the mark". At least it's far more reasonable an assumption than what (E) has us assume: N = N' or "equipment worth the investment" = "equipment better than old".

(C) on the other hand, assumes that "succeed in selling" = "not fail to sell". Isn't that closer to "sound" theories = "not far off the mark" theories?


10 comments

Here's what the NOT flawed version of the stimulus would look like.

(Premise) sound theories AND successful implementation --> lower inflation rate
(Premise) [not] lower inflation rate
___________
(Good conclusion) [not] sound theories AND successful implementation
(Good conclusion with the negation distributed via De Morgan's) not sound theories OR not successful implementation

(Bad conclusion in the stimulus) not sound theories

The argument is flawed because it could be that the theories were fine, just that we sucked at implementing them.

In its abstract form, the flawed argument looks like this:

N and W --> R
/R
___________
/N

(C) matches this form perfectly.

(E) is an attractive wrong answer choice. It's mostly wrong because its logical form does not match:

N --> W and R
/R
___________
/N'

The argument for (E) being better than (C) is that (E) matches the other "mistake" in the argument.

The stimulus argument assumes that "sound" theories = "not far off the mark" theories. True, it does. But, I don't think it's wrong to assume that a "sound" theory is one that's "not far off the mark". At least it's far more reasonable an assumption than what (E) has us assume: N = N' or "equipment worth the investment" = "equipment better than old".

(C) on the other hand, assumes that "succeed in selling" = "not fail to sell". Isn't that closer to "sound" theories = "not far off the mark" theories?


1 comment

Scientists, puzzled about the development of penicillin-resistant bacteria in patients who had not been taking penicillin, believe they have found an explanation. The relevant group of patients have dental fillings made of mercury-containing amalgam, and the bacteria the patients develop are immune to mercury poisoning. Scientists have concluded that the genes causing resistance to penicillin are closely bundled on the chromosomes of bacteria with the gene that produces immunity to mercury poisoning. Exposure to the mercury kills off bacteria that lack the relevant immunity gene, and leaves room for those that possess both the mercury-immunity gene and the penicillin-resistance gene to flourish.

Summarize Argument: Causal Explanation
Scientists are super confused about penicillin-resistant bacteria in patients who don’t use penicillin. (Their confusion implies that resistance normally only occurs when people use penicillin.) However, the scientists have a solution: the bacteria are being naturally selected by mercury instead. This is supported by the bacteria being immune to mercury poisoning. We also learn that mercury-resistance genes are related to penicillin-resistance genes. Finally, the patients in question have mercury in their cavity fillings, giving us a source of mercury exposure.

Identify Argument Part
The assumption that some patients who take penicillin develop bacteria with an immunity to penicillin is part of what makes penicillin resistance without exposure to penicillin confusing.

A
It is a hypothesis that is taken by the scientists to be conclusively proven by the findings described in the passage.
Nothing in the argument is taken to prove a hypothesis that penicillin use can lead to penicillin-resistant bacteria. It’s also not a hypothesis at all, it’s an assumption that’s taken for granted and treated as a fact.
B
It is a generalization that, if true, rules out the possibility that some people who do not take penicillin develop bacteria resistant to it.
The scientists do not believe that penicillin leading to resistance would mean that penicillin is the only way to create resistance. The entire argument is based on the belief that some other factor could lead to resistance, and the scientists just have to figure out what.
C
It is a point that, in conjunction with the fact that some patients who do not take penicillin develop penicillin-resistant bacteria, generates the problem that prompted the research described in the passage.
This is exactly what the assumption about penicillin causing resistance does in the argument. It’s the reason that resistance in the absence of penicillin is confusing, leading to the scientists doing this research in the first place.
D
It is the tentative conclusion of previous research that appears to be falsified by the scientists’ discovery of the mechanism by which bacteria become resistant to mercury poisoning.
Nothing in this argument falsifies anything else. Mercury causing penicillin resistance doesn’t mean that penicillin can’t also cause penicillin resistance. Both are treated as compatible.
E
It is a generalization assumed by the scientists to conclusively prove that the explanation of their problem case must involve reference to the genetic makeup of the penicillin-resistant bacteria.
This is a trap; complicated does not mean correct. The assumption that penicillin can cause resistance doesn’t prove anything. It’s context for why the scientists are confused to start with.

</section


49 comments